Line 233: Line 233:
 
[[Image:QE_11_CS5_1_d_sol2.PNG]]
 
[[Image:QE_11_CS5_1_d_sol2.PNG]]
  
<math>\color{red}
 
\text{Are the cut-offs correct?!}
 
</math>
 
 
----
 
----
 
<math>\color{blue}\text{e) Calculate } y(m,n) \text{ when } x(m,n)=1.</math><br>
 
<math>\color{blue}\text{e) Calculate } y(m,n) \text{ when } x(m,n)=1.</math><br>
Line 264: Line 261:
  
 
<math>\color{red}
 
<math>\color{red}
\text{Is this correct?!}
+
\text{The final answer is correct, but the student has skipped some parts of the derivation and the notations do not sound right.}
 
</math>
 
</math>
  

Revision as of 18:41, 2 August 2012

ECE Ph.D. Qualifying Exam in "Communication, Networks, Signal, and Image Processing" (CS)

Question 5, August 2011, Part 1

Part 1,2]

 $ \color{blue}\text{Consider the following discrete space system with input } x(m,n) \text{ and output } y(m,n). $

                $ \color{blue} y(m,n) = \sum_{k=-\infty}^{\infty}{\sum_{l=-\infty}^{\infty}{x(m-k,n-l)h(k,l)}}. $


$ \color{blue} \text{For parts a) and b) let} $
                $ \color{blue} h(m,n)=sinc(mT,nT) $
$ \color{blue} \text{where } T\leq1. $


$ \color{blue} \text{For parts c), d), and e) let} $
                $ \color{blue} h(m,n)=sinc\left( \frac{(n+m)T}{\sqrt[]{2}},\frac{(n-m)T}{\sqrt[]{2}} \right) $
$ \color{blue} \text{where } T\leq1. $


$ \color{blue}\text{a) Calculate the frequency response, }H \left( e^{j\mu},e^{j\nu} \right). $

$ \color{blue}\text{Solution 1:} $

$ \color{green} \text{Recall:} $

$ \color{green} f(am,bn) \overset{DTFT}{\Leftrightarrow } \frac{1}{|a||b|}F(\frac{\mu}{|a|},\frac{\nu}{|b|}) $

$ \color{green} sinc(m,n) \overset{DTFT}{\Leftrightarrow } rect(\mu,\nu) $

$ H(e^{j\mu},e^{j\nu}) = \frac{1}{T^2} rect(\frac{\mu}{T},\frac{\nu}{T}) $


$ \color{blue}\text{Solution 2:} $

$ sinc(m,n) \rightarrow rect(\mu)rect(\nu) $


$ \Rightarrow sinc(mT,nT) \rightarrow \frac{1}{T^2}rect(\frac{\mu}{T})rect(\frac{\nu}{T}) $

$ \color{green} \text{Here, the student uses the Separability property of the sinc and rect functions.} $


$ \color{blue}\text{b) Sketch the frequency response for } |\mu| < 2\pi \text{ and } |\nu| < 2\pi \text{ when } T = \frac{1}{2} $

$ \color{green} \text{Recall:} $

$ \color{green} rect(t) = \left\{\begin{matrix} 1, for |t|\leq \frac{1}{2} \\ 0, otherwise \end{matrix}\right. $


$ \text{Using the separability property for rect function, for } T = \frac{1}{2} { we have:} $

$ H(e^{j\mu},e^{j\nu}) = \frac{1}{T^2} rect(\frac{\mu}{T},\frac{\nu}{T}) $

$ = 4 rect(2\mu)rect(2\nu) $


QE 11 CS5 1 b.png

$ \text{Note that the gain in this sketch will be } 4. $


$ \color{blue}\text{Solution 2:} $

$ T = \frac{1}{2}, H(e^{j\mu},e^{j\nu}) = 4rect(2\mu)rect(2\nu) $

QE 11 CS5 1 b sol2.PNG



$ \color{blue}\text{c) Calculate the frequency response, }H \left( e^{j\mu},e^{j\nu} \right). $

$ \color{blue}\text{Solution 1:} $

$ \color{green} \text{Recall:} $

$ \color{green} f \left ( A \begin{bmatrix} m \\ n \end{bmatrix} \right) \overset{DTFT}{\Leftrightarrow } \frac{1}{|A|^{-1}}F([\mu, \nu] A^{-1}) $


$ \text{ In this case, A}= \begin{bmatrix} \frac{1}{\sqrt{2}} &\frac{1}{\sqrt{2}} \\ -\frac{1}{\sqrt{2}} &\frac{1}{\sqrt{2}} \end{bmatrix} \text{, hence:} $


$ H(e^{j\mu},e^{j\nu}) = \frac{1}{T^2} rect \left ( \frac{(\mu + \nu)}{\sqrt{2}T},\frac{(\nu - \mu)}{\sqrt{2}T} \right ) $


$ \color{blue}\text{Solution 2:} $

$ \left ( \frac{(n + m)T}{\sqrt{2}},\frac{(n - m)T}{\sqrt{2}} \right) = \begin{bmatrix} \frac{1}{\sqrt{2}} &\frac{1}{\sqrt{2}} \\ -\frac{1}{\sqrt{2}} &\frac{1}{\sqrt{2}} \end{bmatrix} \cdot \begin{pmatrix} mT\\ nT \end{pmatrix} = A \cdot \begin{pmatrix} mT\\ nT \end{pmatrix} $

$ \text{As } |A| = 1, A^{-1} = A^T, sinc \left( A \begin{pmatrix} mT\\ nT \end{pmatrix} \right) \overset{\mathcal{F}}{\rightarrow} F \left( A \begin{pmatrix} \mu\\ \nu \end{pmatrix} \right) $

$ = \frac{1}{T^2} rect \left ( \frac{(\mu + \nu)}{\sqrt{2}T},\frac{(\nu - \mu)}{\sqrt{2}T} \right ) $


$ \color{blue}\text{d) Sketch the frequency response for } |\mu| < 2\pi \text{ and } |\nu| < 2\pi \text{ when } T = \frac{1}{2} $

$ \color{blue}\text{Solution 1:} $

$ \color{green} \text{Recall: Since A is an orthogonal matrix, this transformation is rotationally invariant.} $

$ H(e^{j\mu},e^{j\nu}) = \frac{1}{T^2} rect \left ( \frac{(\mu + \nu)}{\sqrt{2}T},\frac{(\nu - \mu)}{\sqrt{2}T} \right ) $

$ = 4 rect \left (\sqrt{2} (\mu + \nu),\sqrt{2}(\nu - \mu) \right ) $

$ \text{Or} $

$ = 4 rect \left (\sqrt{2} (\mu + \nu) \right) rect \left (\sqrt{2}(\nu - \mu) \right ) $


QE 11 CS5 1 d.PNG

$ \text{Note that the gain in this sketch will be } 4. $



$ \color{blue}\text{Solution 2:} $

$ T = \frac{1}{2}, H(e^{j\mu},e^{j\nu}) = 4rect(\sqrt{2}(\mu + \nu))rect(\sqrt{2}(\nu - \mu)) $

QE 11 CS5 1 d sol2.PNG


$ \color{blue}\text{e) Calculate } y(m,n) \text{ when } x(m,n)=1. $


$ \color{blue}\text{Solution 1:} $


$ Y(e^{j\mu},e^{j\nu}) = \delta(e^{j\mu},e^{j\nu}) \cdot H(e^{j\mu},e^{j\nu}) $

$ = \frac{1}{T^2} rect (0,0) = 4 $

$ \Rightarrow y(m,n) = 4\delta(m,n) $


$ \color{blue}\text{Solution 2:} $

$ y(m,n) = x(m,n) \cdot H(e^{j0},e^{j0}) = 4 $

$ \color{red} \text{The final answer is correct, but the student has skipped some parts of the derivation and the notations do not sound right.} $


"Communication, Networks, Signal, and Image Processing" (CS)- Question 5, August 2011

Go to


Back to ECE Qualifying Exams (QE) page

Alumni Liaison

Ph.D. 2007, working on developing cool imaging technologies for digital cameras, camera phones, and video surveillance cameras.

Buyue Zhang